A 30-year-old woman is experiencing irregular menstrual cycles, and the nurse suspects polycystic ovary syndrome (PCOS). Which of the following symptoms would the nurse expect to see in this patient?

Questions 62

ATI RN

ATI RN Test Bank

Genitourinary System Questions

Question 1 of 5

A 30-year-old woman is experiencing irregular menstrual cycles, and the nurse suspects polycystic ovary syndrome (PCOS). Which of the following symptoms would the nurse expect to see in this patient?

Correct Answer: D

Rationale: The correct answer is D: Excessive weight gain. In PCOS, insulin resistance can lead to weight gain, especially in the abdominal area. This is a common symptom seen in patients with PCOS. Excessive hair growth (Choices A and C) is a symptom of hirsutism, which can occur in PCOS due to hormonal imbalances, but it is not the most specific symptom. Irregular periods and infertility (Choice B) are also common in PCOS, but weight gain is more characteristic and directly linked to the condition.

Question 2 of 5

A 55-year-old woman presents to the clinic with a complaint of severe hot flashes and irregular periods. The nurse suspects perimenopause. Which of the following is the most likely cause of these symptoms?

Correct Answer: A

Rationale: The correct answer is A: Decreased estrogen levels. In perimenopause, the ovaries gradually produce less estrogen, leading to symptoms like hot flashes and irregular periods. Estrogen decline is the primary cause, as evidenced by the characteristic symptoms. Hormonal fluctuations (B) are a result of estrogen decline. Ovarian cysts (C) and increased progesterone levels (D) are less likely causes of perimenopausal symptoms.

Question 3 of 5

Which of the following is true of the vasculature of kidneys?

Correct Answer: B

Rationale: The correct answer is B because blood vessels enter and exit the kidney at the renal hilum, which is where the renal pelvis becomes the ureter. This is important for renal blood supply and urine drainage. A: Renal arteries are posterior to the renal veins, not anterior. C: The left renal artery is shorter and lower than the right renal artery due to the position of the aorta. D: The left renal vein is longer and higher than the right renal vein, as it has a longer course to reach the inferior vena cava.

Question 4 of 5

Filtration of blood occurs in the renal corpuscles. Which of the following is not part of the renal corpuscles?

Correct Answer: B

Rationale: The correct answer is B: Vasa Recta. The Vasa Recta is a network of blood vessels that surrounds the loop of Henle in the kidney medulla, but it is not part of the renal corpuscles. The renal corpuscles consist of the Bowman's capsule, the glomerulus, and the glomerular basement membrane. Bowman's capsule surrounds the glomerulus, and the glomerular basement membrane lies between the Bowman's capsule and the glomerulus, allowing for filtration of blood. Therefore, the Vasa Recta is not part of the renal corpuscles.

Question 5 of 5

A 60-year-old male has come to the clinic for a follow-up visit. Six months ago, he was started on a new medication. The class of medication is most likely to cause impotence as a side effect; therefore, medication classes explored by the nurse are:

Correct Answer: D

Rationale: The correct answer is D, Antihypertensives. Antihypertensive medications are known to commonly cause impotence as a side effect due to their impact on blood flow and circulation. This can affect erectile function in males. Antipyretics (A) are used to reduce fever, bronchodilators (B) are used to treat respiratory conditions, and corticosteroids (C) are anti-inflammatory medications. These classes of medications are not typically associated with impotence as a side effect.

Access More Questions!

ATI RN Basic


$89/ 30 days

ATI RN Premium


$150/ 90 days

Similar Questions